How many numbers in the matrix?Smith Normal Form and lower triangular Toeplitz Matricescriterion for deciding whether the product of a sequence of Givens rotations can reach the full special orthogonal groupHow to write this result (successive Schur complements compose nicely)Partitioned inverse 3x3 block matrixWhat is the criterion for a matrix containing vectors and their permutations being invertible?A linear combination problemHow to find the set of vectors which are as nearly orthogonal as possible?Is this totally unimodular family?How to find eigenvalues of following block matrices?A conjecture about the submatrix of orthogonal matrix

How many numbers in the matrix?


Smith Normal Form and lower triangular Toeplitz Matricescriterion for deciding whether the product of a sequence of Givens rotations can reach the full special orthogonal groupHow to write this result (successive Schur complements compose nicely)Partitioned inverse 3x3 block matrixWhat is the criterion for a matrix containing vectors and their permutations being invertible?A linear combination problemHow to find the set of vectors which are as nearly orthogonal as possible?Is this totally unimodular family?How to find eigenvalues of following block matrices?A conjecture about the submatrix of orthogonal matrix













1












$begingroup$


We consider a matrix $beginbmatrixa_i,jendbmatrix$ with $r$ rows and $c$ columns. We fill this matrix only with zeros and ones.



How many ones (maximally!) we can write into the matrix $rtimes c$ to not have any rectangle with ones at its vertices? Formally:
$$
forall_i_1,i_2in1,ldots,r forall_j_1,j_2in1,ldots,c big( (i_1neq i_2 wedge j_1neq j_2) Rightarrow 0ina_i_1,j_1,a_i_1,j_2, a_i_2,j_1, a_i_2,j_2 big).
$$



For example, for $3times 3$ matrix we can fill it with 6 ones:
$$
beginbmatrix
1 & 1 & 0 \
1 & 0 & 1 \
0 & 1 & 1
endbmatrix.
$$

But when we use 7 ones, then there always is a rectangle with ones at its vertices. For example:
$$beginbmatrix
mathbf 1 & mathbf 1 & 0 \
1 & 0 & 1 \
mathbf 1 & mathbf 1 & 1
endbmatrix.
$$

But what is the maximal number of ones for matrix $rtimes c$?










share|cite|improve this question







New contributor



bonawentura is a new contributor to this site. Take care in asking for clarification, commenting, and answering.
Check out our Code of Conduct.






$endgroup$













  • $begingroup$
    A simple (and not entirely straightforward) argument gives 2(r+c) as a weak upper bound. I suspect the answer is closer to 2max(r,c). (Actually, a tweak gives the latter, which may still be weak.) Gerhard "And What Have You Tried" Paseman, 2019.08.09.
    $endgroup$
    – Gerhard Paseman
    8 hours ago
















1












$begingroup$


We consider a matrix $beginbmatrixa_i,jendbmatrix$ with $r$ rows and $c$ columns. We fill this matrix only with zeros and ones.



How many ones (maximally!) we can write into the matrix $rtimes c$ to not have any rectangle with ones at its vertices? Formally:
$$
forall_i_1,i_2in1,ldots,r forall_j_1,j_2in1,ldots,c big( (i_1neq i_2 wedge j_1neq j_2) Rightarrow 0ina_i_1,j_1,a_i_1,j_2, a_i_2,j_1, a_i_2,j_2 big).
$$



For example, for $3times 3$ matrix we can fill it with 6 ones:
$$
beginbmatrix
1 & 1 & 0 \
1 & 0 & 1 \
0 & 1 & 1
endbmatrix.
$$

But when we use 7 ones, then there always is a rectangle with ones at its vertices. For example:
$$beginbmatrix
mathbf 1 & mathbf 1 & 0 \
1 & 0 & 1 \
mathbf 1 & mathbf 1 & 1
endbmatrix.
$$

But what is the maximal number of ones for matrix $rtimes c$?










share|cite|improve this question







New contributor



bonawentura is a new contributor to this site. Take care in asking for clarification, commenting, and answering.
Check out our Code of Conduct.






$endgroup$













  • $begingroup$
    A simple (and not entirely straightforward) argument gives 2(r+c) as a weak upper bound. I suspect the answer is closer to 2max(r,c). (Actually, a tweak gives the latter, which may still be weak.) Gerhard "And What Have You Tried" Paseman, 2019.08.09.
    $endgroup$
    – Gerhard Paseman
    8 hours ago














1












1








1





$begingroup$


We consider a matrix $beginbmatrixa_i,jendbmatrix$ with $r$ rows and $c$ columns. We fill this matrix only with zeros and ones.



How many ones (maximally!) we can write into the matrix $rtimes c$ to not have any rectangle with ones at its vertices? Formally:
$$
forall_i_1,i_2in1,ldots,r forall_j_1,j_2in1,ldots,c big( (i_1neq i_2 wedge j_1neq j_2) Rightarrow 0ina_i_1,j_1,a_i_1,j_2, a_i_2,j_1, a_i_2,j_2 big).
$$



For example, for $3times 3$ matrix we can fill it with 6 ones:
$$
beginbmatrix
1 & 1 & 0 \
1 & 0 & 1 \
0 & 1 & 1
endbmatrix.
$$

But when we use 7 ones, then there always is a rectangle with ones at its vertices. For example:
$$beginbmatrix
mathbf 1 & mathbf 1 & 0 \
1 & 0 & 1 \
mathbf 1 & mathbf 1 & 1
endbmatrix.
$$

But what is the maximal number of ones for matrix $rtimes c$?










share|cite|improve this question







New contributor



bonawentura is a new contributor to this site. Take care in asking for clarification, commenting, and answering.
Check out our Code of Conduct.






$endgroup$




We consider a matrix $beginbmatrixa_i,jendbmatrix$ with $r$ rows and $c$ columns. We fill this matrix only with zeros and ones.



How many ones (maximally!) we can write into the matrix $rtimes c$ to not have any rectangle with ones at its vertices? Formally:
$$
forall_i_1,i_2in1,ldots,r forall_j_1,j_2in1,ldots,c big( (i_1neq i_2 wedge j_1neq j_2) Rightarrow 0ina_i_1,j_1,a_i_1,j_2, a_i_2,j_1, a_i_2,j_2 big).
$$



For example, for $3times 3$ matrix we can fill it with 6 ones:
$$
beginbmatrix
1 & 1 & 0 \
1 & 0 & 1 \
0 & 1 & 1
endbmatrix.
$$

But when we use 7 ones, then there always is a rectangle with ones at its vertices. For example:
$$beginbmatrix
mathbf 1 & mathbf 1 & 0 \
1 & 0 & 1 \
mathbf 1 & mathbf 1 & 1
endbmatrix.
$$

But what is the maximal number of ones for matrix $rtimes c$?







co.combinatorics matrices






share|cite|improve this question







New contributor



bonawentura is a new contributor to this site. Take care in asking for clarification, commenting, and answering.
Check out our Code of Conduct.










share|cite|improve this question







New contributor



bonawentura is a new contributor to this site. Take care in asking for clarification, commenting, and answering.
Check out our Code of Conduct.








share|cite|improve this question




share|cite|improve this question






New contributor



bonawentura is a new contributor to this site. Take care in asking for clarification, commenting, and answering.
Check out our Code of Conduct.








asked 9 hours ago









bonawenturabonawentura

83 bronze badges




83 bronze badges




New contributor



bonawentura is a new contributor to this site. Take care in asking for clarification, commenting, and answering.
Check out our Code of Conduct.




New contributor




bonawentura is a new contributor to this site. Take care in asking for clarification, commenting, and answering.
Check out our Code of Conduct.
















  • $begingroup$
    A simple (and not entirely straightforward) argument gives 2(r+c) as a weak upper bound. I suspect the answer is closer to 2max(r,c). (Actually, a tweak gives the latter, which may still be weak.) Gerhard "And What Have You Tried" Paseman, 2019.08.09.
    $endgroup$
    – Gerhard Paseman
    8 hours ago

















  • $begingroup$
    A simple (and not entirely straightforward) argument gives 2(r+c) as a weak upper bound. I suspect the answer is closer to 2max(r,c). (Actually, a tweak gives the latter, which may still be weak.) Gerhard "And What Have You Tried" Paseman, 2019.08.09.
    $endgroup$
    – Gerhard Paseman
    8 hours ago
















$begingroup$
A simple (and not entirely straightforward) argument gives 2(r+c) as a weak upper bound. I suspect the answer is closer to 2max(r,c). (Actually, a tweak gives the latter, which may still be weak.) Gerhard "And What Have You Tried" Paseman, 2019.08.09.
$endgroup$
– Gerhard Paseman
8 hours ago





$begingroup$
A simple (and not entirely straightforward) argument gives 2(r+c) as a weak upper bound. I suspect the answer is closer to 2max(r,c). (Actually, a tweak gives the latter, which may still be weak.) Gerhard "And What Have You Tried" Paseman, 2019.08.09.
$endgroup$
– Gerhard Paseman
8 hours ago











2 Answers
2






active

oldest

votes


















5












$begingroup$

It seems you just want to maximise the number of ones so that there is no
$$left[matrix1&1\1&1right]$$
as a not-necessarily-contiguous $2times 2$ submatrix.



Equivalently, how many edges can a bipartite graph with sides of size $r$ and $s$ have without having any 4-cycles?



This is the first non-trivial case of the Zarankiewicz problem.



The square case is A001197(n)-1.



There is a table of values and bounds in this paper. I have more exact values near the diagonal.






share|cite|improve this answer











$endgroup$














  • $begingroup$
    In particular, see OEIS sequences A191873 and A001197.
    $endgroup$
    – Robert Israel
    8 hours ago










  • $begingroup$
    Also A072567.
    $endgroup$
    – Rob Pratt
    7 hours ago



















1












$begingroup$

For an $n times n$ matrix, you can get $3 (n-1)$ ones by putting them in all entries except $(1,1)$ of the first row, first column and main diagonal. That's optimal at least for $n le 5$.



But for $6 times 6$ the optimal solution has $16$ ones, e.g.



$$ left[ begin arraycccccc 0&1&1&1&0&0\ 0&1&0&0
&1&1\ 1&1&0&0&0&0\ 1&0&1&0&1&0
\ 1&0&0&1&0&1\ 0&0&0&1&1&0
end array right]
$$






share|cite|improve this answer











$endgroup$

















    Your Answer








    StackExchange.ready(function()
    var channelOptions =
    tags: "".split(" "),
    id: "504"
    ;
    initTagRenderer("".split(" "), "".split(" "), channelOptions);

    StackExchange.using("externalEditor", function()
    // Have to fire editor after snippets, if snippets enabled
    if (StackExchange.settings.snippets.snippetsEnabled)
    StackExchange.using("snippets", function()
    createEditor();
    );

    else
    createEditor();

    );

    function createEditor()
    StackExchange.prepareEditor(
    heartbeatType: 'answer',
    autoActivateHeartbeat: false,
    convertImagesToLinks: true,
    noModals: true,
    showLowRepImageUploadWarning: true,
    reputationToPostImages: 10,
    bindNavPrevention: true,
    postfix: "",
    imageUploader:
    brandingHtml: "Powered by u003ca class="icon-imgur-white" href="https://imgur.com/"u003eu003c/au003e",
    contentPolicyHtml: "User contributions licensed under u003ca href="https://creativecommons.org/licenses/by-sa/3.0/"u003ecc by-sa 3.0 with attribution requiredu003c/au003e u003ca href="https://stackoverflow.com/legal/content-policy"u003e(content policy)u003c/au003e",
    allowUrls: true
    ,
    noCode: true, onDemand: true,
    discardSelector: ".discard-answer"
    ,immediatelyShowMarkdownHelp:true
    );



    );






    bonawentura is a new contributor. Be nice, and check out our Code of Conduct.









    draft saved

    draft discarded


















    StackExchange.ready(
    function ()
    StackExchange.openid.initPostLogin('.new-post-login', 'https%3a%2f%2fmathoverflow.net%2fquestions%2f338028%2fhow-many-numbers-in-the-matrix%23new-answer', 'question_page');

    );

    Post as a guest















    Required, but never shown

























    2 Answers
    2






    active

    oldest

    votes








    2 Answers
    2






    active

    oldest

    votes









    active

    oldest

    votes






    active

    oldest

    votes









    5












    $begingroup$

    It seems you just want to maximise the number of ones so that there is no
    $$left[matrix1&1\1&1right]$$
    as a not-necessarily-contiguous $2times 2$ submatrix.



    Equivalently, how many edges can a bipartite graph with sides of size $r$ and $s$ have without having any 4-cycles?



    This is the first non-trivial case of the Zarankiewicz problem.



    The square case is A001197(n)-1.



    There is a table of values and bounds in this paper. I have more exact values near the diagonal.






    share|cite|improve this answer











    $endgroup$














    • $begingroup$
      In particular, see OEIS sequences A191873 and A001197.
      $endgroup$
      – Robert Israel
      8 hours ago










    • $begingroup$
      Also A072567.
      $endgroup$
      – Rob Pratt
      7 hours ago
















    5












    $begingroup$

    It seems you just want to maximise the number of ones so that there is no
    $$left[matrix1&1\1&1right]$$
    as a not-necessarily-contiguous $2times 2$ submatrix.



    Equivalently, how many edges can a bipartite graph with sides of size $r$ and $s$ have without having any 4-cycles?



    This is the first non-trivial case of the Zarankiewicz problem.



    The square case is A001197(n)-1.



    There is a table of values and bounds in this paper. I have more exact values near the diagonal.






    share|cite|improve this answer











    $endgroup$














    • $begingroup$
      In particular, see OEIS sequences A191873 and A001197.
      $endgroup$
      – Robert Israel
      8 hours ago










    • $begingroup$
      Also A072567.
      $endgroup$
      – Rob Pratt
      7 hours ago














    5












    5








    5





    $begingroup$

    It seems you just want to maximise the number of ones so that there is no
    $$left[matrix1&1\1&1right]$$
    as a not-necessarily-contiguous $2times 2$ submatrix.



    Equivalently, how many edges can a bipartite graph with sides of size $r$ and $s$ have without having any 4-cycles?



    This is the first non-trivial case of the Zarankiewicz problem.



    The square case is A001197(n)-1.



    There is a table of values and bounds in this paper. I have more exact values near the diagonal.






    share|cite|improve this answer











    $endgroup$



    It seems you just want to maximise the number of ones so that there is no
    $$left[matrix1&1\1&1right]$$
    as a not-necessarily-contiguous $2times 2$ submatrix.



    Equivalently, how many edges can a bipartite graph with sides of size $r$ and $s$ have without having any 4-cycles?



    This is the first non-trivial case of the Zarankiewicz problem.



    The square case is A001197(n)-1.



    There is a table of values and bounds in this paper. I have more exact values near the diagonal.







    share|cite|improve this answer














    share|cite|improve this answer



    share|cite|improve this answer








    edited 8 hours ago

























    answered 8 hours ago









    Brendan McKayBrendan McKay

    26.6k1 gold badge54 silver badges109 bronze badges




    26.6k1 gold badge54 silver badges109 bronze badges














    • $begingroup$
      In particular, see OEIS sequences A191873 and A001197.
      $endgroup$
      – Robert Israel
      8 hours ago










    • $begingroup$
      Also A072567.
      $endgroup$
      – Rob Pratt
      7 hours ago

















    • $begingroup$
      In particular, see OEIS sequences A191873 and A001197.
      $endgroup$
      – Robert Israel
      8 hours ago










    • $begingroup$
      Also A072567.
      $endgroup$
      – Rob Pratt
      7 hours ago
















    $begingroup$
    In particular, see OEIS sequences A191873 and A001197.
    $endgroup$
    – Robert Israel
    8 hours ago




    $begingroup$
    In particular, see OEIS sequences A191873 and A001197.
    $endgroup$
    – Robert Israel
    8 hours ago












    $begingroup$
    Also A072567.
    $endgroup$
    – Rob Pratt
    7 hours ago





    $begingroup$
    Also A072567.
    $endgroup$
    – Rob Pratt
    7 hours ago












    1












    $begingroup$

    For an $n times n$ matrix, you can get $3 (n-1)$ ones by putting them in all entries except $(1,1)$ of the first row, first column and main diagonal. That's optimal at least for $n le 5$.



    But for $6 times 6$ the optimal solution has $16$ ones, e.g.



    $$ left[ begin arraycccccc 0&1&1&1&0&0\ 0&1&0&0
    &1&1\ 1&1&0&0&0&0\ 1&0&1&0&1&0
    \ 1&0&0&1&0&1\ 0&0&0&1&1&0
    end array right]
    $$






    share|cite|improve this answer











    $endgroup$



















      1












      $begingroup$

      For an $n times n$ matrix, you can get $3 (n-1)$ ones by putting them in all entries except $(1,1)$ of the first row, first column and main diagonal. That's optimal at least for $n le 5$.



      But for $6 times 6$ the optimal solution has $16$ ones, e.g.



      $$ left[ begin arraycccccc 0&1&1&1&0&0\ 0&1&0&0
      &1&1\ 1&1&0&0&0&0\ 1&0&1&0&1&0
      \ 1&0&0&1&0&1\ 0&0&0&1&1&0
      end array right]
      $$






      share|cite|improve this answer











      $endgroup$

















        1












        1








        1





        $begingroup$

        For an $n times n$ matrix, you can get $3 (n-1)$ ones by putting them in all entries except $(1,1)$ of the first row, first column and main diagonal. That's optimal at least for $n le 5$.



        But for $6 times 6$ the optimal solution has $16$ ones, e.g.



        $$ left[ begin arraycccccc 0&1&1&1&0&0\ 0&1&0&0
        &1&1\ 1&1&0&0&0&0\ 1&0&1&0&1&0
        \ 1&0&0&1&0&1\ 0&0&0&1&1&0
        end array right]
        $$






        share|cite|improve this answer











        $endgroup$



        For an $n times n$ matrix, you can get $3 (n-1)$ ones by putting them in all entries except $(1,1)$ of the first row, first column and main diagonal. That's optimal at least for $n le 5$.



        But for $6 times 6$ the optimal solution has $16$ ones, e.g.



        $$ left[ begin arraycccccc 0&1&1&1&0&0\ 0&1&0&0
        &1&1\ 1&1&0&0&0&0\ 1&0&1&0&1&0
        \ 1&0&0&1&0&1\ 0&0&0&1&1&0
        end array right]
        $$







        share|cite|improve this answer














        share|cite|improve this answer



        share|cite|improve this answer








        edited 8 hours ago

























        answered 8 hours ago









        Robert IsraelRobert Israel

        45.2k54 silver badges125 bronze badges




        45.2k54 silver badges125 bronze badges























            bonawentura is a new contributor. Be nice, and check out our Code of Conduct.









            draft saved

            draft discarded


















            bonawentura is a new contributor. Be nice, and check out our Code of Conduct.












            bonawentura is a new contributor. Be nice, and check out our Code of Conduct.











            bonawentura is a new contributor. Be nice, and check out our Code of Conduct.














            Thanks for contributing an answer to MathOverflow!


            • Please be sure to answer the question. Provide details and share your research!

            But avoid


            • Asking for help, clarification, or responding to other answers.

            • Making statements based on opinion; back them up with references or personal experience.

            Use MathJax to format equations. MathJax reference.


            To learn more, see our tips on writing great answers.




            draft saved


            draft discarded














            StackExchange.ready(
            function ()
            StackExchange.openid.initPostLogin('.new-post-login', 'https%3a%2f%2fmathoverflow.net%2fquestions%2f338028%2fhow-many-numbers-in-the-matrix%23new-answer', 'question_page');

            );

            Post as a guest















            Required, but never shown





















































            Required, but never shown














            Required, but never shown












            Required, but never shown







            Required, but never shown

































            Required, but never shown














            Required, but never shown












            Required, but never shown







            Required, but never shown







            Popular posts from this blog

            Invision Community Contents History See also References External links Navigation menuProprietaryinvisioncommunity.comIPS Community ForumsIPS Community Forumsthis blog entry"License Changes, IP.Board 3.4, and the Future""Interview -- Matt Mecham of Ibforums""CEO Invision Power Board, Matt Mecham Is a Liar, Thief!"IPB License Explanation 1.3, 1.3.1, 2.0, and 2.1ArchivedSecurity Fixes, Updates And Enhancements For IPB 1.3.1Archived"New Demo Accounts - Invision Power Services"the original"New Default Skin"the original"Invision Power Board 3.0.0 and Applications Released"the original"Archived copy"the original"Perpetual licenses being done away with""Release Notes - Invision Power Services""Introducing: IPS Community Suite 4!"Invision Community Release Notes

            Canceling a color specificationRandomly assigning color to Graphics3D objects?Default color for Filling in Mathematica 9Coloring specific elements of sets with a prime modified order in an array plotHow to pick a color differing significantly from the colors already in a given color list?Detection of the text colorColor numbers based on their valueCan color schemes for use with ColorData include opacity specification?My dynamic color schemes

            Tom Holland Mục lục Đầu đời và giáo dục | Sự nghiệp | Cuộc sống cá nhân | Phim tham gia | Giải thưởng và đề cử | Chú thích | Liên kết ngoài | Trình đơn chuyển hướngProfile“Person Details for Thomas Stanley Holland, "England and Wales Birth Registration Index, 1837-2008" — FamilySearch.org”"Meet Tom Holland... the 16-year-old star of The Impossible""Schoolboy actor Tom Holland finds himself in Oscar contention for role in tsunami drama"“Naomi Watts on the Prince William and Harry's reaction to her film about the late Princess Diana”lưu trữ"Holland and Pflueger Are West End's Two New 'Billy Elliots'""I'm so envious of my son, the movie star! British writer Dominic Holland's spent 20 years trying to crack Hollywood - but he's been beaten to it by a very unlikely rival"“Richard and Margaret Povey of Jersey, Channel Islands, UK: Information about Thomas Stanley Holland”"Tom Holland to play Billy Elliot""New Billy Elliot leaving the garage"Billy Elliot the Musical - Tom Holland - Billy"A Tale of four Billys: Tom Holland""The Feel Good Factor""Thames Christian College schoolboys join Myleene Klass for The Feelgood Factor""Government launches £600,000 arts bursaries pilot""BILLY's Chapman, Holland, Gardner & Jackson-Keen Visit Prime Minister""Elton John 'blown away' by Billy Elliot fifth birthday" (video with John's interview and fragments of Holland's performance)"First News interviews Arrietty's Tom Holland"“33rd Critics' Circle Film Awards winners”“National Board of Review Current Awards”Bản gốc"Ron Howard Whaling Tale 'In The Heart Of The Sea' Casts Tom Holland"“'Spider-Man' Finds Tom Holland to Star as New Web-Slinger”lưu trữ“Captain America: Civil War (2016)”“Film Review: ‘Captain America: Civil War’”lưu trữ“‘Captain America: Civil War’ review: Choose your own avenger”lưu trữ“The Lost City of Z reviews”“Sony Pictures and Marvel Studios Find Their 'Spider-Man' Star and Director”“‘Mary Magdalene’, ‘Current War’ & ‘Wind River’ Get 2017 Release Dates From Weinstein”“Lionsgate Unleashing Daisy Ridley & Tom Holland Starrer ‘Chaos Walking’ In Cannes”“PTA's 'Master' Leads Chicago Film Critics Nominations, UPDATED: Houston and Indiana Critics Nominations”“Nominaciones Goya 2013 Telecinco Cinema – ENG”“Jameson Empire Film Awards: Martin Freeman wins best actor for performance in The Hobbit”“34th Annual Young Artist Awards”Bản gốc“Teen Choice Awards 2016—Captain America: Civil War Leads Second Wave of Nominations”“BAFTA Film Award Nominations: ‘La La Land’ Leads Race”“Saturn Awards Nominations 2017: 'Rogue One,' 'Walking Dead' Lead”Tom HollandTom HollandTom HollandTom Hollandmedia.gettyimages.comWorldCat Identities300279794no20130442900000 0004 0355 42791085670554170004732cb16706349t(data)XX5557367